LSAT and Law School Admissions Forum

Get expert LSAT preparation and law school admissions advice from PowerScore Test Preparation.

User avatar
 Dave Killoran
PowerScore Staff
  • PowerScore Staff
  • Posts: 5853
  • Joined: Mar 25, 2011
|
#45449
Complete Question Explanation
(The complete setup for this game can be found here: lsat/viewtopic.php?t=2450)

The correct answer choice is (B)

If G drives on Monday, then J must drive on Wednesday. According to answer choice (B), G drives on Monday and J drives on Tuesday. But, this creates a situation where J drives on both Tuesday and Wednesday, a violation of the second rule. Consequently, answer choice (B) cannot be true and is correct.

Get the most out of your LSAT Prep Plus subscription.

Analyze and track your performance with our Testing and Analytics Package.